LSAT and Law School Admissions Forum

Get expert LSAT preparation and law school admissions advice from PowerScore Test Preparation.

User avatar
 Dave Killoran
PowerScore Staff
  • PowerScore Staff
  • Posts: 5852
  • Joined: Mar 25, 2011
|
#26966
Complete Question Explanation

The correct answer choice is (A)

This problem can be solved by looking solely at seat 2:

Answer choice (B) is incorrect because O’s seat is in the last row.

Answer choice (C) is incorrect because we established in the setup that P’s seat was seat 5.

Answer choice (D) is incorrect because R’s seat cannot be in the first row (R must sit in seat 4 or 6).

Answer choice (E) is incorrect because U cannot be assigned to seat 2.

Accordingly, answer choice (A) is proven correct by process of elimination.
 DJ93
  • Posts: 2
  • Joined: Apr 05, 2017
|
#33929
Hi there,

So I am not quite sure why answers B and E don't work, because they also could be true. Right? Oh wait, the answers are given in the format of (2,8) .... I think I figured it out! Thanks! :-D
 Robert Carroll
PowerScore Staff
  • PowerScore Staff
  • Posts: 1787
  • Joined: Dec 06, 2013
|
#33960
DJ,

Yep, "respectively" means that each answer will list what's supposed to be in seat 2, then what's supposed to be in seat 8, in that order. So answer choice (B) is wrong because O cannot be in the first row (and seat 2 is in the first row!), and answer choice (E) is wrong because U cannot be in seat 2 (that would cause problems with N).

Robert Carroll
 alissajacques
  • Posts: 1
  • Joined: Apr 22, 2020
|
#75054
Hello!

Quick question. In the setup, N and R are a vertical block. If N is in 2 then R would be in 5. But only P can be in 5. Would you be able to explain what I might be missing?

Thank you!
User avatar
 KelseyWoods
PowerScore Staff
  • PowerScore Staff
  • Posts: 1079
  • Joined: Jun 26, 2013
|
#75084
Hi Alissa!

Careful with how you interpret that NR rule! Here's the segment from the full Setup and Rule Diagram Explanation post (https://forum.powerscore.com/lsat/viewtopic.php?t=2291):
Administrator wrote:Setup and Rule Diagram Explanation

Note carefully that the third rule states that R’s seat is the row behind the row in which N’s seat is located. The rule does not say that R’s seat is the seat behind N’s seat. This rule is often misinterpreted by test takers. Remember, always read closely!
So if N is in seat 2, R would still be in seat 4 or 6.

Hope this helps!

Best,
Kelsey

Get the most out of your LSAT Prep Plus subscription.

Analyze and track your performance with our Testing and Analytics Package.